LSAT and Law School Admissions Forum

Get expert LSAT preparation and law school admissions advice from PowerScore Test Preparation.

 Administrator
PowerScore Staff
  • PowerScore Staff
  • Posts: 8916
  • Joined: Feb 02, 2011
|
#23500
Complete Question Explanation

Strengthen. The correct answer choice is (B)

Part of the difficulty in the problem is that "central Glenwood" and "central Redville" are left undefined. It could be that they are similar in size, or significantly different in size. For example, central Chicago is much larger than central San Diego, and this library might not be in walking distance for central Chicago residents, even if the library is in central Chicago. Therefore, in order to strengthen the argument, we will likely need to find the answer choice that addresses that potential difference in size.

Answer Choice (A): This answer choice is irrelevant to the argument that putting a library in central Glenwood would put it within walking distance of more residents.

Answer Choice (B): This is the correct answer choice.
This answer acknowledges the potential differences in size of the two communities and dismisses the possibility that central Greenwood could be substantially larger than central Redville.

Answer Choice (C): The size of the building has nothing to do with the argument regarding the number of people who would be within walking distance of the new library location. Nothing in the argument addresses the issue of whether the new library will be better or worse than any other options for the library, only how many people will have walking access to the library.

Answer Choice (D): The argument also has nothing to do with how many people overall use the library or would be potentially inconvenienced by it moving. The argument is only concerned with increasing the number of people within walking distance of the library.

Answer Choice (E): This answer choice would, if anything, weaken the argument by indicating that people walk currently walk further than what is generally considered walking distance to get to the library. This would seem to indicate that more people could be considered within the current library's walking distance than the stimulus seems to imply.
 angie23
  • Posts: 25
  • Joined: Nov 17, 2013
|
#15113
I do not understand how E) hurts the argument. Originally, I thought it was correct because the answer choice shows how the current library location is within walking distance for a smaller number of library users than what would happen if it relocated.

In addition to that, I didn't think that B) strengthen that much. Even if central Glenwood is the same size as central Redville, the library's new location might be still within walking distance of a smaller or same number of library users, since people who live on the outskirts of central Glenwood might still be considered living within walking distances. Can someone help sort through my reasoning? Thanks!
 Lucas Moreau
PowerScore Staff
  • PowerScore Staff
  • Posts: 216
  • Joined: Dec 13, 2012
|
#15122
Hello, angie,

E doesn't necessarily hurt the argument so much as it is not especially relevant to the argument. The argument concerns itself with how many people will be within walking distance, not how long that walking distance should be compared to reality.

The conclusion is that moving the library to central Glenwood will put it within walking distance of more library users. E might strengthen the conclusion of there being more people ultimately at the library, but it doesn't strengthen the link between the premise of there being more people in central Glenwood and the conclusion of more people vising the library.

B is correct, because even if central Glenwood has twice as many people as central Redville, if central Glenwood is spread out over ten times as much surface area, there will most likely be ultimately less people within walking distance of the library in central Glenwood than there are now in central Redville.

Hope that helps,
Lucas Moreau
 linda.an
  • Posts: 9
  • Joined: Jun 08, 2016
|
#26332
The stimulus says that if the public library is relocated to central Glenwood, "the library would then be within walking distance of a larger number of library users."

So why does the size of central Glenwood and the size of central Revile matter when we know that more library users will be able to walk to the library after relocation?

Please help me out, thank you!
User avatar
 Jonathan Evans
PowerScore Staff
  • PowerScore Staff
  • Posts: 726
  • Joined: Jun 09, 2016
|
#26377
Linda, good question that addresses the heart of issues that involve helping (or hurting) an argument. After you work a stimulus and notice that there is an argument involved, i.e. that there is a conclusion and some supporting premises, you should ask yourself whether the argument is sound or unsound. Does it seem like the conclusion is a sure bet or are you skeptical for some reason? Maybe you are slightly skeptical but not quite sure what's wrong. In this case you notice in the question stem that the task is to strengthen the argument. OK now you know that there's something wrong with the argument or that for some reason the conclusion may not be totally airtight. This "problem" or "flaw" in the argument is the means whereby you need to help out the conclusion.

Look and see that the conclusion is that if the library is moved from Central Redville to Central Glenwood, it will now be within walking distance of a larger number of users. This is the main point, and the truth or falsehood of this statement is in question. Since you want to help out the conclusion, you need to help establish that this conclusion is indeed a good one, that by moving the library, the new location will indeed be within walking distance of a larger number of users.

So what supporting evidence do we have for this claim. To begin, we know the library is located currently in Central Redville. We also know there are more people in Central Glenwood than in Central Redville. We also know people will walk to the library only if it is located close to their homes (if ppl walk to a library :arrow: library is close to their homes). The author thus concludes that by moving the library to the more populous Central Glenwood, more people will be within walking distance.

Here's the problem, if we're to make a conclusion about how many people will be within walking distance in one place versus another place, we need to know how many people are present in a given area.

The author pulls a trick on us here. We are switching from one area (Central Redville) to another (Central Glenwood). Whenever there is a comparison between properties of two things, you need to make sure those two things are analogous in every possible way.

What if Central Redville had only 500 ppl but was only one square mile in size? Now imagine Central Glenwood has 1000 ppl but is 6 square miles in size. Even though Central Glenwood has more people, do we know for sure that these people are as concentrated together in Central Glenwood as they are in Central Redville? No we don't! Do we know that more people will be within walking distance? No we don't!

That's the trick. One way to strengthen this conclusion is to establish that the two "Central" areas are roughly the same size. Therefore, we know that if you have more people in an area that is roughly the same size as an old area, the number of people within walking distance will likely increase.

Thus answer choice (B) is the credited response.

I hope this helps. If you would like to discuss other answer choices or require further explanation, please reply and let me know!
 AnnBar
  • Posts: 33
  • Joined: Mar 24, 2017
|
#35452
Hello,

I am trying to understand answer choices B and E. I chose answer choice E because I figured if the distance people currently walk is farther than what is generally considered walking distance then there is a high likelihood others will also be willing to walk further than what is generally considered walking distance, and as a result more people would be going to the new library location.

After reading the explanation, I want to see if I am understanding the reasoning behind answer B. The are covered by central Glendwood is approximately the same size as that covered by central Redville, which means same surface area, but more people in Glendwood =higher likelihood people will be at a closer distance from the library?

Thank you
 Ricky_Hutchens
PowerScore Staff
  • PowerScore Staff
  • Posts: 59
  • Joined: Oct 12, 2015
|
#35565
Hi AnnBar,

Your understanding is spot on. If central Redville is the same size as central Glenwood, but there are more people in central Glenwood, then we can expect there to be more people close to the library if it is moved from central Redvill to central Glenwood.

As to your originally assumption, if people are currently walking farther than what is generally considered walking distance, doesn't necessarily mean other people will do so. There could be reasons people walk farther in Redville than they do else where.
 AnnBar
  • Posts: 33
  • Joined: Mar 24, 2017
|
#35615
Thank you!
 Toby
  • Posts: 33
  • Joined: Jun 05, 2017
|
#35718
Hi!

I have two questions. First, I am a bit confused by the explanations I have read for this question. I am using the "Logical Reasoning Question Type Training Volume 1: LSAT PrepTests 1-20" book to study the "Weaken" question types, and in my book, the question stem for this question is :

"Which one of the following, if true, most seriously weakens the argument?"

I see that many of the explanations of this question address it as though it is a "Strengthen" question. Would someone mind explaining this discrepancy to me?

My second question has to do with the answer that I selected. I chose letter D because I thought this one would effectively weaken the conclusion. I thought the conclusion was about being able to walk to the library, so I letter D because I thought it showed that regardless of the library's location, most people will not choose to walk to the library. I'm not quite sure why letter B weakens the argument. Thank you so much for your help!
 Francis O'Rourke
PowerScore Staff
  • PowerScore Staff
  • Posts: 471
  • Joined: Mar 10, 2017
|
#35801
Hi Toby,

It looks like you are referring to question #11 from this test. Questions number 10 and 11 refer to the same stimulus, so they are easy to confuse.

Number 10 was a strengthen question, while number 11 was a weaken question. You can find discussion of the latter question here.

Get the most out of your LSAT Prep Plus subscription.

Analyze and track your performance with our Testing and Analytics Package.